AAMC CBT3 and 3R OFFICIAL Q&A

This forum made possible through the generous support of SDN members, donors, and sponsors. Thank you.

Vihsadas

No summer
Moderator Emeritus
Lifetime Donor
10+ Year Member
5+ Year Member
15+ Year Member
Joined
Oct 17, 2007
Messages
5,474
Reaction score
56
This is the official Q&A thread for AAMC CBT3 and 3R.

Please post ONLY questions pertaining to AAMC CBT3 and 3R.
Out of respect for people who may not have completed the other exams, do not post questions or material from any other AAMC exam.

Please see this thread for the rules of order before you post.

Good luck on your MCAT!

Members don't see this ad.
 
Q #17:

This question is about Heating Acetamide (mp = 80'C) at 90'C vs 100'C. I am stuck between (A) More water would have been needed to melt the acetamide vs. (D) The acetamide would have taken longer to completely melt.

Are we to assume that the temperature of the water STAYS at 90'C? Or do we bring it to 90'C and remove the heating source? b/c that could make a difference.

If the latter please tell me if the following logic is wrong:

q = mc(delta)T so shouldn't lower T necessitate a higher m to melt the acetamide?

Also thinking along these lines from a different perspective, lets say you barely cover the test tube with water @ 90'C, since you have so little water and so much acetamide equilibrium will be attained at a temperature very close to that of the acetamide. VERSUS if you had a **** ton of water at 90'C you would get closer to 90'C and be able to melt acetamide. (Think of jumping into a bathtub @ 90'C vs. a swimming pool @ 90'C. In the former you would equilibrate very fast to your original temperature versus the swimming pool would probably cause major degree burns...)
 
Q #17:

This question is about Heating Acetamide (mp = 80'C) at 90'C vs 100'C. I am stuck between (A) More water would have been needed to melt the acetamide vs. (D) The acetamide would have taken longer to completely melt.

Are we to assume that the temperature of the water STAYS at 90'C? Or do we bring it to 90'C and remove the heating source? b/c that could make a difference.

If the latter please tell me if the following logic is wrong:

q = mc(delta)T so shouldn't lower T necessitate a higher m to melt the acetamide?

Also thinking along these lines from a different perspective, lets say you barely cover the test tube with water @ 90'C, since you have so little water and so much acetamide equilibrium will be attained at a temperature very close to that of the acetamide. VERSUS if you had a **** ton of water at 90'C you would get closer to 90'C and be able to melt acetamide. (Think of jumping into a bathtub @ 90'C vs. a swimming pool @ 90'C. In the former you would equilibrate very fast to your original temperature versus the swimming pool would probably cause major degree burns...)

but isn't the m here referring to the mass of acetamide? since we are checking the energy required to melt acetamide.

The energy comes from the heated water but the mass of interest is the mass of acetamide.

please someone correct me if I'm wrong.
 
Q lost = Q gained.

I'm referring to the Q lost - in which case m would refer to mass of the water.
 
Members don't see this ad :)
For verbal #55, Roman number 2 says "fewer law enforcement officials would be retained" What does "retained" mean? They'd be fired? Or would they be reassigned?

Retained means "to keep" so fewer officials would need to be kept, because the legalization of drugs would be beneficial (according to the passage) and officials wouldn't have to constantly go after illegal drug possession.

For my question:
TLDR: Does "total" voltage mean the sum of the voltages of each device in a series?

Thus, if the question asks how the total voltage of a circuit with one device compares to the total voltage of a circuit with two identical devices connected in series, are the total voltages the same?

"Suppose that two identical photoelectric devices are connected to the same electrical circuit, and the devices are connected in parallel with each other. Compared to the voltage generated by a single device, the total voltage supplied to the circuit by the parallel devices will be":

the same because they're parallel. And the total current supplied to the circuit will be twice as big (because resistance is half as big).

However, what if the question were switched to two identical devices in series with each other: "compared to the voltage generated by a single device, the total voltage supplied to the circuit by the devices in series will be:

I get that with two identical devices in series, each device will have half of the voltage compared to a circuit with only one device, but the total voltage supplied by the devices will still be the same, wouldn't it, b/c they would add up? Or would this question be asking for the voltage supplied by one of the devices in the series? Because I got confused by the bolded part in their answer, which I have copied below. (And total current supplied by the two devices would be the half as the current supplied by one device, though the current in two devices would be the same)

The answer says "identical voltage sources connected in parallel produce the same output voltage as a single source. (Whereas if they were connected in series, the source voltages would be added" <-- I'm confused by the bolded part. I think what they mean by "total" is the addition of the two voltages, which would equal the original voltage supplied by just one device?
 
Retained means "to keep" so fewer officials would need to be kept, because the legalization of drugs would be beneficial (according to the passage) and officials wouldn't have to constantly go after illegal drug possession.

For my question:
TLDR: Does "total" voltage mean the sum of the voltages of each device in a series?

Thus, if the question asks how the total voltage of a circuit with one device compares to the total voltage of a circuit with two identical devices connected in series, are the total voltages the same?

"Suppose that two identical photoelectric devices are connected to the same electrical circuit, and the devices are connected in parallel with each other. Compared to the voltage generated by a single device, the total voltage supplied to the circuit by the parallel devices will be":

the same because they're parallel. And the total current supplied to the circuit will be twice as big (because resistance is half as big).

However, what if the question were switched to two identical devices in series with each other: "compared to the voltage generated by a single device, the total voltage supplied to the circuit by the devices in series will be:

I get that with two identical devices in series, each device will have half of the voltage compared to a circuit with only one device, but the total voltage supplied by the devices will still be the same, wouldn't it, b/c they would add up? Or would this question be asking for the voltage supplied by one of the devices in the series? Because I got confused by the bolded part in their answer, which I have copied below. (And total current supplied by the two devices would be the half as the current supplied by one device, though the current in two devices would be the same)

The answer says "identical voltage sources connected in parallel produce the same output voltage as a single source. (Whereas if they were connected in series, the source voltages would be added" <-- I'm confused by the bolded part. I think what they mean by "total" is the addition of the two voltages, which would equal the original voltage supplied by just one device?

2 Batteries in series -> total voltage is sum of the 2 battery voltages
2 same V batteries in parallel -> total voltage is just one of them as in the q
 
For number 30, it asks..
2 identical voltage sources are connected in parallel. Compared to voltage generated by a single source, how will the new voltage differ?
The answer is that the voltage is the same.
How does batteries in parallel have the same voltage as one? Is there a formula for equivalence voltage in parallel?

I feel so dumb right now, I completely misread your question :/ My verbal score deserves to be lower.
 
Last edited:
2 Batteries in series -> total voltage is sum of the 2 battery voltages
2 same V batteries in parallel -> total voltage is just one of them as in the q

OMGGG and here I was thinking that "device" means resistor :/ DANG. If it's a battery, then that makes sense. Thanks!
 
OMGGG and here I was thinking that "device" means resistor :/ DANG. If it's a battery, then that makes sense. Thanks!
The rules for series and parallel I was wondering if you can clarify something for me.
Series: Rt=R1+R2+R3 C=1/C1+1/C2 +1/C3 what about i and V. I remember one of the stays the same the other doesn't but I do not have my notes on me.
 
The rules for series and parallel I was wondering if you can clarify something for me.
Series: Rt=R1+R2+R3 C=1/C1+1/C2 +1/C3 what about i and V. I remember one of the stays the same the other doesn't but I do not have my notes on me.
Series: Voltage is added, I is contstant. Parallel: V constant, I is added.
 
In PS #15 you have a rectangle 4m wide by 3m long, one force of 4N parallel to the 4m at the corner and another force of 3N pointing parallel to the 3m side. They ask you to find the force on a uknown vector to ensure translational equilibrium.

The answer explanation didn't take into consideration the distance of the 3N and 4N vector from the center (where the unknown vector originates from) Why is that? Don't we have to determine the energy lost by placing the force vectors far on the edge of the rectangle?

Yes, can someone please explain this question? The solution completely ignored the angle theta which the force was applied and just applied pythagoreans theorem.
 
Yes, can someone please explain this question? The solution completely ignored the angle theta which the force was applied and just applied pythagoreans theorem.

Because the tangent of the angle is unnecessary to solve the problem. All you need to do is balancing the forces so that both horizontal and vertical forces cancel out. You would apply the same amount of force in opposite direction, and the sum of the horizontal and vertical forces can be found by the pythagorean theorem.
 
Because the tangent of the angle is unnecessary to solve the problem. All you need to do is balancing the forces so that both horizontal and vertical forces cancel out. You would apply the same amount of force in opposite direction, and the sum of the horizontal and vertical forces can be found by the pythagorean theorem.

Hmm, are you sure? If the angel was zero translational equilibrium would be impossible. This angle has to be relevant right?
 
Members don't see this ad :)
Ok then since the tangent value does not indicate that the angle is 0 or 90. Translational equilibrium is possible, so you just take the sum of horizontal and vertical force.

When I first saw that question, I played around with the tangent value, but couldn't find the answer until my insight told me that there's a simpler way of solving the problem which is mentioned above.
 
I think I see.

So really, there is only ONE possible force and angle combination that can make this happen... If Im picturing this right. That angle tells us nothing except that this is do-able. If it was any other angle, rotational equilibrium would be impossible regardless of the applied force, correct?
 
Yeah you're right in that there's only one possible force (magnitude and direction) that can result in the translational equilibrium given the condition.
 
This problem threw me off at first. All you have to do is add the vectors to create a vector in the opposite direction of the force and solve for it.
 
I had a question on #38 which asks what will happen to post-menopausal women receiving estrogen/progesterone therapies.

The answer is: periodic menstruation will occur.

How can this be? After the LH spike in women who are pre-menopausal, levels of estrogen and progesterone spike up to inhibit LH- this is important because it prevents the development of future follicles if the egg is successfully fertilized (which is why the corpus leuteum and placenta both produce these two hormones till term)

So if you were to give a women these hormones, their menstruation should stop shouldn't it?

Can someone please explain this one for me too? I was under the impression that Progesterone/Estrogen was administered in birth control pills to prevent future follicles and menstruation from happening!
 
Not too sure, but isn't it because estrogen/progesterone will thicken the endometrium and when the hormones are gone, (assuming they're given in accordance with the hormonal cycles of pre-menopausal women) the endometrium will slough off?
 
Thats essentially what the AAMC explanation said. But how and why does estrogen/progesterone behave differently in this context than in a premenopausal woman taking estrogen/progesterone birth control pills?
 
Ok I looked up some Wiki about estrogen. You know how, before ovulation, estrogen and progesterone negatively feedsback on LH and FSH? This is how oral contraceptive works. By giving doses of estrogen, progesterone, LH is suppressed (which delays ovulation) and FSH is suppresed (which delays follicle maturation.)

In post menopausal women, their hormone levels are lower than pre menopausal women. (I think the psg talks about estrogen withdrawal symptom.) So given doses will get the women back on the cycling level of hormone->menstruation. I doubt that they'll be ovulating though. I am not sure on this.
 
just scored 10 6 10.

****. Most of it was content review too. Verbal also burnt me out. Tips on how to not get bogged down? That cornelius passage ruined me.
 
Those dashed waves do not have equal amplitudes.

4OOB956.gif
 
  • Like
Reactions: 1 user
Hey, everyone. Don't mean to derail the thread, but was wondering about AAMC MCAT CBT 3 Passage 1 Question 4. I don't understand why SO4 is more soluble than I which is more soluble than CO3? Can someone explain?

Why is it that whatever is the most soluble precipitates first?
 
Also, on AAMC MCAT CBT 3 Passage 2 Question 8, why do we do n1 - n2, instead of the other way around? I understand we cannot have a negative energy difference. However, I am concerned that a future question will have a smaller n1 and a larger n2, and that I will absentmindedly do n2-n1...instead of the proper n1-n2. Can anyone answer why we subtract n1-n2 (instead of n2-n1)?
 
Not sure if this has already been asked, but can some one explain Item 15 for me? It's Item 15 from the PS section:

A rectangular sheet of material has a width of 3 m and a length of 4 m. Forces with magnitudes of 3 N n 4 N, respectively, are applied parallel to two edges of the sheet, as shown in the figure below.

figure_zps88dc245b.png


A third force F, is applied to the center of the sheet, along a line in the plane of the sheet, at an angle theta = arctan 0.75 with respect to the horizontal direction. The sheet will be in translational equilibrium when F has what value?


I understand that in order for the sheet to be in translational motion, the net external forces have to equal 0, but I'm confused about how to arrive at the answer exactly. Can anyone provide a clear explanation as to how to arrive at the answer? Much appreciated, since my MCAT is on Saturday!

Thanks!
 
A rectangular sheet of material has a width of 3 m and a length of 4 m. Forces with magnitudes of 3 N n 4 N, respectively, are applied parallel to two edges of the sheet, as shown in the figure below.

figure_zps88dc245b.png


A third force F, is applied to the center of the sheet, along a line in the plane of the sheet, at an angle theta = arctan 0.75 with respect to the horizontal direction. The sheet will be in translational equilibrium when F has what value?

----------------
So we are in translational equilibrium when the components of all external forces cancel. PLEASE someone correct me if I'm wrong, but I solved for the forces in regards to torque=F*l (Torque = Force times lever arm because its a rotating force.)

t=(4)(4) =16
t=(3)(3)=9
Gives us a total torque of 25 so counterbalancing force is t=F*l = sqrt(25) = 5 N.
IS THIS CORRECT???
 
Last edited:
it is asking about translational equilibrium; torque is for rotation equilibrium. you are trying to find the magnitude of F such that Fx = 4 N and Fy= 3 N.
 
A rectangular sheet of material has a width of 3 m and a length of 4 m. Forces with magnitudes of 3 N n 4 N, respectively, are applied parallel to two edges of the sheet, as shown in the figure below.

figure_zps88dc245b.png


A third force F, is applied to the center of the sheet, along a line in the plane of the sheet, at an angle theta = arctan 0.75 with respect to the horizontal direction. The sheet will be in translational equilibrium when F has what value?

----------------
So we are in translational equilibrium when the components of all external forces cancel. PLEASE someone correct me if I'm wrong, but I solved for the forces in regards to torque=F*l (Torque = Force times lever arm because its a rotating force.)

t=(4)(4) =16
t=(3)(3)=9
Gives us a total torque of 25 so counterbalancing force is t=F*l = sqrt(25) = 5 N.
IS THIS CORRECT???


No. I don't see where torque is involved in this. Where is the rotation/pivot? I think the fact they emphasize *translational* equilibrium should hint that you shouldn't even think about torque.

The quick way of solving this is 1) realizing that the force F is pointed in the angle but in the opposite direction as the net force caused by the 4N and 3N forces. To sum those forces, make a right triangle in your head by putting the tail of the 3N vector to the head of the 4N vector. The net force is 5N pointing diagonally down (recognize the 3-4-5 triangle).

Now in order to have equilibrium, the force F needs to be pointing completely opposite the net 5N vector we just calculated. You can just assume the angles of F in the diagram work out, because the answer choices are just magnitudes. E.g., if you consider choice D, 7N...well there is no way a 7N force could equally oppose a 5N force without creating a new force at a different angle. For example 7*cos(theta) might equal 5, and oppose the 5N force, but there is a new force now in the sin(theta) direction.

But if you don't feel comfortable just assuming the angles "work out", they help you out by giving you the arctan of theta. Since tan is equal to the ratio of opposite/adjacent, and this ratio is 3/4 = .75, you can see that the little triangle created by the F vector with angle theta is similar to the right triangle we envisioned earlier. And since it's similar, it means the angle of F is the same as the angle of the 5N force we mentioned. So you pick 5N because it's a 5N force in the direction opposite the one created by the 3N and 4N forces summed.
 
  • Like
Reactions: 1 user
For #35 the oxalic acid question, why is it C? It said the pH was 4.7, which is below the pka of one of the COOHs for oxalic acid. This means that that COOH is still protonated, which means that it DIDN'T dissociate and therefore only 1 COOH dissociated (thus 2 particles in solution). Crotonic acid dissociates into 2 particles too.

Therefore, I don't see why oxalic acid would dissociate into 3 particles and thus why the correct answer is C?

Thanks
 
For #35 the oxalic acid question, why is it C? It said the pH was 4.7, which is below the pka of one of the COOHs for oxalic acid. This means that that COOH is still protonated, which means that it DIDN'T dissociate and therefore only 1 COOH dissociated (thus 2 particles in solution). Crotonic acid dissociates into 2 particles too.

Therefore, I don't see why oxalic acid would dissociate into 3 particles and thus why the correct answer is C?

Thanks

No, this is only true before the first equivalence point (I understand it is difficult to know where exactly it is, but that is how it is on the MCAT). At the point where the pH=pka2 (4.77) this is where HOOCCOO- = -OOCCOO-. Notice I stated that the concentrations are equal. Meaning that even if the pH is somewhat lower, there will still be double de-protonated oxalic acids (-OOCCOO-) floating around. What will end up in the solution at pH=4.7 is more single de-protonated oxalic acids than double. This still means that oxalic acid has more ions. Hope this helps.
 
Last edited:
No, this is only true before the first equivalence point (I understand it is difficult to know where exactly it is, but that is how it is on the MCAT). At the point where the pH=pka2 (4.77) this is where HOOCCOO- = -OOCCOO-. Notice I stated that the concentrations are equal. Meaning that even if the pH is somewhat lower, there will still be double de-protonated oxalic acids (-OOCCOO-) floating around. What will end up in the solution at pH=4.7 is more single de-protonated oxalic acids than double. This still means that oxalic acid has more ions. Hope this helps.

Thanks! Your explanation makes a lot of sense. The only thing I'm not quite sure is what you mean by "this is only true before the first equivalence point"?
 
Also, for Bio question #102 (HC cholesterol passage), why is HC codominant? I was kind of confused by their explanation. It seems like it would fit more with incomplete dominance, even though that's not an answer choice.
 
Thanks! Your explanation makes a lot of sense. The only thing I'm not quite sure is what you mean by "this is only true before the first equivalence point"?

What I meant by that was that the oxalic acid is in its single de-protonated form (HOOCOO-) "completely" at the equivalence point (which is between pka1 and pka2 on the weak acid titration curve). Once it goes beyond that point it starts to double de-protonated (-OOCOO-).

Also, for Bio question #102 (HC cholesterol passage), why is HC codominant? I was kind of confused by their explanation. It seems like it would fit more with incomplete dominance, even though that's not an answer choice.

It is kind of a difficult thing to understand and explain. The way incomplete dominance works is that it is a blend of the phenotype. So if a red flower (dominant) is crossed with a white one (recessive), then you get a pink flower (intermediate). In this case of co-dominance, both phenotypes are being expressed independently of each other. So with the blood types, both A antigens (dominant) and B antigens (also dominant) are being expressed completely (100%), without a mixture of the two. Thus a person with AB blood will have A and B antigens as separate units on their blood cells. This is what co-dominance is.

So in the case of HC, the dominant mutant allele prevents the expression of the LDL receptor and the dominant wild type allele allows for expression of the LDL receptors. What is happening here is that the heterozygous individual is expressing only 50% of the receptors because one of the wild type alleles is working, thus only half of the receptors are being made. The other half of production is being stopped by the mutant allele. There is no blend of the two here, they are acting separately. One controls the production of 50% of the receptors and the other controls the other 50%. If the condition were dominant over recessive, then all 100% of the receptors would be produced if the wild type allele were dominant.

Hope this helps
 
What I meant by that was that the oxalic acid is in its single de-protonated form (HOOCOO-) "completely" at the equivalence point (which is between pka1 and pka2 on the weak acid titration curve). Once it goes beyond that point it starts to double de-protonated (-OOCOO-).



It is kind of a difficult thing to understand and explain. The way incomplete dominance works is that it is a blend of the phenotype. So if a red flower (dominant) is crossed with a white one (recessive), then you get a pink flower (intermediate). In this case of co-dominance, both phenotypes are being expressed independently of each other. So with the blood types, both A antigens (dominant) and B antigens (also dominant) are being expressed completely (100%), without a mixture of the two. Thus a person with AB blood will have A and B antigens as separate units on their blood cells. This is what co-dominance is.

So in the case of HC, the dominant mutant allele prevents the expression of the LDL receptor and the dominant wild type allele allows for expression of the LDL receptors. What is happening here is that the heterozygous individual is expressing only 50% of the receptors because one of the wild type alleles is working, thus only half of the receptors are being made. The other half of production is being stopped by the mutant allele. There is no blend of the two here, they are acting separately. One controls the production of 50% of the receptors and the other controls the other 50%. If the condition were dominant over recessive, then all 100% of the receptors would be produced if the wild type allele were dominant.

Hope this helps
Yes this helps a lot! Thanks!!
 
Question 132. A man is treated with low doses of an estrogen analogue to destroy and estrogen-responsive adrenal tumor. Compared to an age-matched control (no estrogen treatment), this patient's chances of developing osteoporosis will most likely be:

A) increased
B) decreased
C) approximately the same
D) approximately the same, but the disease will appear at an earlier age


Can someone please explain why C is the correct answer?
 
Question 132. A man is treated with low doses of an estrogen analogue to destroy and estrogen-responsive adrenal tumor. Compared to an age-matched control (no estrogen treatment), this patient's chances of developing osteoporosis will most likely be:

A) increased
B) decreased
C) approximately the same
D) approximately the same, but the disease will appear at an earlier age


Can someone please explain why C is the correct answer?

This is one of those "make the link with far off information in the passage on top off making far of deductions that you will never get in hell" type question.

1) The beginning of the passage states post-menopausal woman lose bone mass 8-10 years after menopause (osteoporosis).
2) Now, you have to link this statement to the one made in the last paragraph. They state that hormone replacement is used to prevent osteoporosis in women.
3) Now. you have to link this to the next sentence. "Because estrogen therapy may increase risk of developing uterine or breast cancer..."
4) So you now know that estrogen is used for women that have osteoporosis during the post-menopausal period.
5) The next inference is that estrogen is most likely low in post-menopasual women with osteoporosis, hence the reason you give them estrogen as a therapy in the first place.
6) Now you have to make one final inference. "This has no effect on osteoporosis in men because the whole thing has to do with post-menopasual women only."
7) This is why C is correct.

Well I got this one wrong also, so yeah it was that hard.
 
  • Like
Reactions: 1 user
This is one of those "make the link with far off information in the passage on top off making far of deductions that you will never get in hell" type question.

1) The beginning of the passage states post-menopausal woman lose bone mass 8-10 years after menopause (osteoporosis).
2) Now, you have to link this statement to the one made in the last paragraph. They state that hormone replacement is used to prevent osteoporosis in women.
3) Now. you have to link this to the next sentence. "Because estrogen therapy may increase risk of developing uterine or breast cancer..."
4) So you now know that estrogen is used for women that have osteoporosis during the post-menopausal period.
5) The next inference is that estrogen is most likely low in post-menopasual women with osteoporosis, hence the reason you give them estrogen as a therapy in the first place.
6) Now you have to make one final inference. "This has no effect on osteoporosis in men because the whole thing has to do with post-menopasual women only."
7) This is why C is correct.

Well I got this one wrong also, so yeah it was that hard.

I hate this question... Thanks though!
 
Can someone please help me with #17 in the PS section. The question is about which of the following would most likely have occurred if the water had only reached 90 C before the test tube was placed into it.
The two choices I narrowed it down to were:
A) More water would have been needed to melt the acetamide
D) the Acetamide would have taken longer to completely melt.

Could someone please explain why D Is the correct answer? I knew it wasn't B or C, so but I incorrectly chose A. The AAMC explanation isn't really clearing things up for me so I am still confused.
if q_gained=q_lost can't an increase in m_water make up for a smaller delta T? Also, how can the process of melting taking longer at lower temperatures be justified?
Thanks.
 
Can someone please help me with #17 in the PS section. The question is about which of the following would most likely have occurred if the water had only reached 90 C before the test tube was placed into it.
The two choices I narrowed it down to were:
A) More water would have been needed to melt the acetamide
D) the Acetamide would have taken longer to completely melt.

Could someone please explain why D Is the correct answer? I knew it wasn't B or C, so but I incorrectly chose A. The AAMC explanation isn't really clearing things up for me so I am still confused.
if q_gained=q_lost can't an increase in m_water make up for a smaller delta T? Also, how can the process of melting taking longer at lower temperatures be justified?
Thanks.

I am not 100% sure if my reasoning is correct but when I chose D because temperature is the average energy of the solution. If you decrease the temperature there is less energy and therefore it will take longer for it to heat up.
 
Can someone please help me with #17 in the PS section. The question is about which of the following would most likely have occurred if the water had only reached 90 C before the test tube was placed into it.
The two choices I narrowed it down to were:
A) More water would have been needed to melt the acetamide
D) the Acetamide would have taken longer to completely melt.

Could someone please explain why D Is the correct answer? I knew it wasn't B or C, so but I incorrectly chose A. The AAMC explanation isn't really clearing things up for me so I am still confused.
if q_gained=q_lost can't an increase in m_water make up for a smaller delta T? Also, how can the process of melting taking longer at lower temperatures be justified?
Thanks.

I am not 100% sure if my reasoning is correct but when I chose D because temperature is the average energy of the solution. If you decrease the temperature there is less energy and therefore it will take longer for it to heat up.

The rate of heat transfer is known as thermal or heat flux. Just like other gradients, the greater the gradient, the greater the driving force to bring the system to equilibrium. The rate of heat transfer is given by the equation:

7ec5031759804c8278b3c3097e3b6fdb.png


Basically, if Q is the same and we decrease the temperature gradient, then the time it takes for that amount of heat to transfer must increase.
 
  • Like
Reactions: 1 user
Hello, I'm having issues with number 112.

The solution to question 110, which talks about what would improve the degree f separation between limonene and (+) carvone talks about how using a vacuum source that achieves a lower pressure inside the distillation apparatus will "lower the boiling points of both liquids and narrow their difference in boiling point."

Okay now question 112

If a leak develops in the vacuum distillation apparatus, the boiling points of the two components of caraway seed oil will:
a. both increase
b. both decrease
c. both remain the same
d. become more similar

So I get A, because of the atmospheric pressure and such. But the solution to 110 is now really confusing me as to why D would not be the correct answer to this question. And why they would even become more similar to begin with.

Thanks.
 
Hello, I'm having issues with number 112.

The solution to question 110, which talks about what would improve the degree f separation between limonene and (+) carvone talks about how using a vacuum source that achieves a lower pressure inside the distillation apparatus will "lower the boiling points of both liquids and narrow their difference in boiling point."

Okay now question 112

If a leak develops in the vacuum distillation apparatus, the boiling points of the two components of caraway seed oil will:
a. both increase
b. both decrease
c. both remain the same
d. become more similar

So I get A, because of the atmospheric pressure and such. But the solution to 110 is now really confusing me as to why D would not be the correct answer to this question. And why they would even become more similar to begin with.

Thanks.

I think D is a distractor type question. It is like saying the boiling point of one will increase while the other one will decrease, thus becoming more similar.
 
I am confused about number 104 (BS passage 2).

The questions goes: Which of the following statements provides the strongest support for the hypothesis that HC is a genetic disease rather than a disease caused by environmental factors?
A. HC is relatively common in some families and absent from others.
B. There is no significant difference in the dietary habits of individuals in affected families versus individuals in unaffected families.
C. HC appears to be caused by a malfunction at the plasma membrane.
D. The cells of healthy individuals have cholesterol-containing endocytotic vesicles, whereas the cells of individuals with HC do not.

The answer is B, but I picked D. I understand the B definitely supports the hypothesis that HC is genetic, but wouldn't D support it even better? A difference in diet doesn't necessarily account for all factors in a family's lifestyle that could affect serum cholesterol, but the lack of a vesicle--which the passage says is the result of a lack of a particular protein--would, to me, point directly to a genetic disorder. I would be more apt to believe this disease is genetically related knowing that, at the molecular level, a protein is missing, than merely observing that there appears to be no difference in diet. The only explanation for how a protein could be completely missing from an entire family would be that there's a genetic abnormality, no?

(Edit: One might point out that type 2 diabetes is the result of the downregulation of insulin receptors, and thus a family with poor eating habits will all have fewer insulin receptor proteins in their cell membranes. But even in the most chronic stages of type 2 diabetes, the insulin receptors still exist, even if at very small amounts. This passage said that people with HC outright lack a necessary protein; they're not just deficient in it.)

Appreciate anyone's input.
 
Last edited:
I am confused about number 104 (BS passage 2).

The questions goes: Which of the following statements provides the strongest support for the hypothesis that HC is a genetic disease rather than a disease caused by environmental factors?
A. HC is relatively common in some families and absent from others.
B. There is no significant difference in the dietary habits of individuals in affected families versus individuals in unaffected families.
C. HC appears to be caused by a malfunction at the plasma membrane.
D. The cells of healthy individuals have cholesterol-containing endocytotic vesicles, whereas the cells of individuals with HC do not.

The answer is B, but I picked D. I understand the B definitely supports the hypothesis that HC is genetic, but wouldn't D support it even better? A difference in diet doesn't necessarily account for all factors in a family's lifestyle that could affect serum cholesterol, but the lack of a vesicle--which the passage says is the result of a lack of a particular protein--would, to me, point directly to a genetic disorder. I would be more apt to believe this disease is genetically related knowing that, at the molecular level, a protein is missing, than merely observing that there appears to be no difference in diet. The only explanation for how a protein could be completely missing from an entire family would be that there's a genetic abnormality, no?

(Edit: One might point out that type 2 diabetes is the result of the downregulation of insulin receptors, and thus a family with poor eating habits will all have fewer insulin receptor proteins in their cell membranes. But even in the most chronic stages of type 2 diabetes, the insulin receptors still exist, even if at very small amounts. This passage said that people with HC outright lack a necessary protein; they're not just deficient in it.)

Appreciate anyone's input.

This is a classic case of just overthinking the question. (Don't worry, I did it too!)

Think about this in simple terms. The passage talks all about hypercholesterolemia and all the different genetic factors that scientists have researched to find out what could be causing it. They also casually mention that there is no difference in dietary habits of families studied.

So we have two variables here: genetic and environmental. When we look at this question, we have all biological factors we're looking at essentially. Eliminate A because it's vague. C and D are both biological, but I personally picked D because it was a more specific biological causation. In my post-game, I realized that there's one very simple factor that the test-makers want you to realize here. Picking C - you eliminate any chances of HC being an environmental disease. Think about it. If there is NO SIGNIFICANT DIETARY difference, there is no chance that this can be caused environmentally. It HAS to be genetic. So regardless of whatever crazy biological factor they isolate, nothing can trump them disproving the other variable.

Does that make sense? This is not a case of D being incorrect, C is just a better answer. The tipoff should have been the fact that they gave two answer choices giving seemingly comparable biological mechanisms that could've explained a genetic causation.
 
Last edited by a moderator:
  • Like
Reactions: 1 user
This is a classic case of just overthinking the question. (Don't worry, I did it too!)

Think about this in simple terms. The passage talks all about hypercholesterolemia and all the different genetic factors that scientists have researched to find out what could be causing it. They also casually mention that there is no difference in dietary habits of families studied.

So we have two variables here: genetic and environmental. When we look at this question, we have all biological factors we're looking at essentially. Eliminate A because it's vague. C and D are both biological, but I personally picked D because it was a more specific biological causation. In my post-game, I realized that there's one very simple factor that the test-makers want you to realize here. Picking C - you eliminate any chances of HC being an environmental disease. Think about it. If there is NO SIGNIFICANT DIETARY difference, there is no chance that this can be caused environmentally. It HAS to be genetic. So regardless of whatever crazy biological factor they isolate, nothing can trump them disproving the other variable.

Does that make sense? This is not a case of D being incorrect, C is just a better answer. The tipoff should have been the fact that they gave two answer choices giving seemingly comparable biological mechanisms that could've explained a genetic causation.
Yeah, I suppose that makes sense. I dunno, I think I'd still contest their reasoning from an academic standpoint... but from an MCAT standpoint, I suppose they're always right, haha.
 
Question 132. A man is treated with low doses of an estrogen analogue to destroy and estrogen-responsive adrenal tumor. Compared to an age-matched control (no estrogen treatment), this patient's chances of developing osteoporosis will most likely be:

A) increased
B) decreased
C) approximately the same
D) approximately the same, but the disease will appear at an earlier age


Can someone please explain why C is the correct answer?

estrogen is dangerous at high levels and often has to be subsided by a dose of progesterone but since it's a low estrogen it really has no impact. If it was a female with high levels of estrogen without progesterone, you are basically asking for high cell turnover aka cancer.
 
Top